Finding Volume of Solid Bounded by Equations: How to Setup a Double Integral?

In summary, the conversation outlines a problem involving finding the volume of a solid bounded by the graphs of two equations. The equations are x^2+z^2=1, y^2+z^2=1, z=sqrt(1-x^2), and z=sqrt(1-y^2). The individual discusses their approach to setting up a double integral to solve the problem, and raises some questions about the chosen limits and the final answer. After some discussion, it is determined that there was a slight error in the original approach, and the correct solution involves multiplying the integral by two and limiting the range of z to the smaller of the two values, sqrt(1-x^2) and sqrt(1-y^2).
  • #1
mugzieee
77
0
i have to setup a doble integral to find the volume of the solid bounded by the graphs of the equation.

x^2+z^2=1, and y^2+z^2=1
z=sqrt(1-x^2)
z=sqrt(1-y^2)

then substituting in z=sqrt(1-y^2) into x^2+z^2=1, i got y=x.

so when i setup a double integral
for the dy i get integral of(sqrt(1-y^2)) and the limits are from 0-x
and for the limits of he dx i get 0-1.

the way i got the lim its of x was by solving the equation x^2+z^2=1 for x by making z=0, but I am not sure if that's right.

and finally when i integrate my double integral, I am getting a wrong answer.
 
Physics news on Phys.org
  • #2
I'll reply to this with what I can help you with since no one else has replied yet. I remember posting a very similar problem a while ago but I've forgotten all about it now. :rolleyes: The following is just what I can come up with right now so its probably best to wait for further replies. I would also suggest including the answer which you seem to have.

When doing these sorts of problems I think you need to view projections wrt a single plane - for example yhe yz plane or the xy plane. This is just arbitrary but I would 'project' everything onto the xz plane. So you can 'ignore' the equation x^2 + z^2 = 1 for now. You also have y^2 + z^2 = 1 where y is the 'vertical' part of the coordinate system since the projection is onto the xz plane. Solving for y yields y = +/-sqrt(1-z^2).

[tex]
V = \int\limits_{ - 1}^1 {\int\limits_{ - \sqrt {1 - x^2 } }^{\sqrt {1 - x^2 } } {\int\limits_{ - \sqrt {1 - z^2 } }^{\sqrt {1 - z^2 } } {dydzdx} } }
[/tex]

Note: I checked on mathworldhttp://mathworld.wolfram.com/SteinmetzSolid.html) and the volume is given by the integral above. I checked it so that I could verify my answer. Although I have little idea as to how this why the limits are the way that they are. I mean, if I had to guess I would've gone with those limits but I just don't understand them. The limits seem like they would only work if the cylinders intersected in perfect circles but this doesn't appear to be the case.

mugizee - don't worry about the paragraph above this, those are just things that have been troubling me. Also, you see that I have used a triple integral but it really just reduces to a double integral if that's what you really need to use. This is because the integrand is equal to one and so after the first integration it's equivalent to starting off with a double integral. I used a triple integral because I prefer to use them for volumes.

So [tex]V = \int\limits_{ - 1}^1 {\int\limits_{ - \sqrt {1 - x^2 } }^{\sqrt {1 - x^2 } } {\int\limits_{ - \sqrt {1 - z^2 } }^{\sqrt {1 - z^2 } } {dydzdx} } }[/tex]

[tex]
= \int\limits_0^{2\pi } {\int\limits_{ - \sqrt {1 - \sin ^2 \theta } }^{\sqrt {1 - \sin ^2 \theta } } {\int\limits_{ - \sqrt {1 - \cos ^2 \theta } }^{\sqrt {1 - \cos ^2 \theta } } {rdydrd\theta } } }
[/tex]...where x = sin(theta) and z = cos(theta), assigned this way because the y-axis is the 'vertical' axis wrt to the chosen projection

[tex]
= \int\limits_0^{2\pi } {\int\limits_{ - \cos \theta }^{\cos \theta } {\int\limits_{ - \sin \theta }^{\sin \theta } {rdydrd\theta } } }
[/tex]

[tex]
= 2\int\limits_0^{2\pi } {\int\limits_{ - \cos \theta }^{\cos \theta } r } \sin \theta drd\theta
[/tex]

[tex]
= 2\int\limits_0^{2\pi } {\cos ^2 \theta \sin \theta d\theta }
[/tex]

[tex]
= - \frac{2}{3}\left[ {\cos ^3 \theta } \right]_0^{2\pi }
[/tex]

Ok there's an error somewhere since the 'answer' comes out to be zero. It should be 16/3. Perhaps someone else could give you some help with this because I can't get the answer.
 
Last edited by a moderator:
  • #3
thanx for tryn Benny, but unfortanately the answer you gave was incorrect. the way the book does it is like this:
inner integral= sqrt(1-x^2)dy,0,x
outer integals limits are 0-1, dx
and the entire integral multiplied by two.

the final answer is 2/3

the equation they gave for the xy plane was simply y=x.

what i don't uderstand is why they chose the sqrt(1-x^2) equation for the z-equation, why couldn't they choose the other z=equation, or woul it have been the same? And why did they multiply the entire integral by two?
 
  • #4
Hmm that's strange because the answer I got from mathworld is 16/3, perhaps I misread one of their answers. You can check if you want to, the relevant page is the link that I provided in my first post.

At the moment I just can't see why the book's solution works/doesn't work. Hopefully someone else can give you some help with this one. Sorry... :redface:
 
  • #5
mugzieee said:
thanx for tryn Benny, but unfortanately the answer you gave was incorrect. the way the book does it is like this:
inner integral= sqrt(1-x^2)dy,0,x
outer integals limits are 0-1, dx
and the entire integral multiplied by two.

the final answer is 2/3

the equation they gave for the xy plane was simply y=x.

what i don't uderstand is why they chose the sqrt(1-x^2) equation for the z-equation, why couldn't they choose the other z=equation, or woul it have been the same? And why did they multiply the entire integral by two?

Benny's answer is correct for the entire region bounded by two intersecting cylinders. Your answer is correct for the region bounded by the intersecting cylinders and the planes y = 0 and y = x. Your integral is confined to the region y <= x in the first quadrant, which is one eighth of the region contained by the intersecting cylinders. Within your region of integration, z is limited by the smaller of the two values sqrt(1-x^2) and sqrt(1-y^2), which would be sqrt(1-x^2) since y <= x. The integral is multiplied by two because the range of z is symmetric about z = 0; the height of a volume element dV = zdxdy is 2*sqrt(1-x^2).
 
Last edited:
  • #6
thanks, i get it now. that was a tricky problem...
 

1. What is a double integral?

A double integral is a type of integral that involves integrating a function of two variables over a region in a two-dimensional plane. It is used to find the volume under a surface in a three-dimensional space, as well as other applications in physics, engineering, and mathematics.

2. How is a double integral set up?

A double integral is set up by first identifying the region of integration, which is typically a rectangle or a more complex shape. Then, the function to be integrated is written as a function of two variables, and the limits of integration are determined for each variable.

3. What is the difference between a single integral and a double integral?

A single integral involves integrating a function of one variable over a one-dimensional interval, while a double integral involves integrating a function of two variables over a two-dimensional region. In other words, a single integral finds the area under a curve, while a double integral finds the volume under a surface.

4. Why is a double integral useful?

A double integral is useful because it allows us to calculate the volume and other properties of three-dimensional objects, which have many real-world applications. It also helps in solving various problems in physics, engineering, and economics.

5. What are the different types of double integrals?

There are two types of double integrals: iterated and double integrals. An iterated integral is a double integral that is expressed as a sequence of two single integrals, with the inner integral being evaluated first. A double integral, on the other hand, is evaluated as a single integral using the Fubini's theorem.

Similar threads

  • Introductory Physics Homework Help
Replies
9
Views
920
  • Introductory Physics Homework Help
2
Replies
64
Views
2K
  • Introductory Physics Homework Help
Replies
25
Views
1K
  • Introductory Physics Homework Help
Replies
1
Views
663
  • Introductory Physics Homework Help
Replies
9
Views
129
  • Introductory Physics Homework Help
Replies
13
Views
730
  • Introductory Physics Homework Help
Replies
6
Views
1K
  • Introductory Physics Homework Help
Replies
3
Views
1K
  • Introductory Physics Homework Help
Replies
15
Views
261
  • Introductory Physics Homework Help
Replies
19
Views
672
Back
Top